K
Khách

Hãy nhập câu hỏi của bạn vào đây, nếu là tài khoản VIP, bạn sẽ được ưu tiên trả lời.

AH
Akai Haruma
Giáo viên
5 tháng 3 2018

Thay \(x+y+z=12\) thì:

\(M=\frac{x+12-15}{x}+\frac{y+12-15}{y}+\frac{z+12-15}{z}\)

\(M=\frac{x-3}{x}+\frac{y-3}{y}+\frac{z-3}{z}=1-\frac{3}{x}+1-\frac{3}{y}+1-\frac{3}{z}\)

\(M=3-3\left(\frac{1}{x}+\frac{1}{y}+\frac{1}{z}\right)\)

Với điều kiện trên của $x,y,z$ thì biểu thức M có max thôi em nhé.

5 tháng 3 2018

\(M=\dfrac{2x+y+z-15}{x}+\dfrac{x+2y+z-15}{y}+\dfrac{x+y+2z-15}{z}\)

\(M=\dfrac{x+\left(x+y+z\right)-15}{x}+\dfrac{y+\left(x+y+z\right)-15}{y}+\dfrac{z+\left(x+y+z\right)-15}{z}\)\(M=\dfrac{x-3}{x}+\dfrac{y-3}{y}+\dfrac{z-3}{z}\)

\(\dfrac{3-M}{3}=\dfrac{1}{x}+\dfrac{1}{y}+\dfrac{1}{z}\) cần tìm max \(\dfrac{1}{x}+\dfrac{1}{y}+\dfrac{1}{z}=N\)

c/m không tồn tại N_max

trong 3 số (x;y;z) chỉ cần một số tiến đến 0 ; N-->vô cùng

Đề có sai không bạn?

ko làm đc thì chắc là sai thôi bạn hiha

NV
14 tháng 1 2021

\(T=\dfrac{\left(xy\right)^2}{zx+zy}+\dfrac{\left(yz\right)^2}{xy+xz}+\dfrac{\left(zx\right)^2}{yx+yz}\ge\dfrac{xy+yz+zx}{2}\ge\dfrac{3}{2}\sqrt[3]{\left(xyz\right)^2}=\dfrac{3}{2}\)

 

28 tháng 1 2018

\(M=\frac{2x+y+z-15}{x}+\frac{x+2y+z-15}{y}+\frac{x+y+2z-15}{z}\)

\(M-3=\frac{x+y+z-15}{x}+\frac{x+y+z-15}{y}+\frac{x+y+z-15}{z}\)

\(M-3=\left(x+y+z-15\right)\left(\frac{1}{x}+\frac{1}{y}+\frac{1}{z}\right)\)

\(\Rightarrow M\ge\left(x+y+z-15\right)\cdot\frac{9}{x+y+z}+3=\frac{3}{4}\)

\("="\Leftrightarrow x=y=z=4\)

26 tháng 1 2018

nhận ra là bài này sai đề :)))

NV
28 tháng 3 2023

Chắc đề là \(x+y+z=3\)

Ta có: 

\(\left(2x+y+z\right)^2=\left(x+y+x+z\right)^2\ge4\left(x+y\right)\left(x+z\right)\)

\(\Rightarrow P\le\dfrac{x}{4\left(x+y\right)\left(x+z\right)}+\dfrac{y}{4\left(x+y\right)\left(y+z\right)}+\dfrac{z}{4\left(x+z\right)\left(y+z\right)}\)

\(\Rightarrow P\le\dfrac{x\left(y+z\right)+y\left(z+x\right)+z\left(x+y\right)}{4\left(x+y\right)\left(y+z\right)\left(z+x\right)}=\dfrac{xy+yz+zx}{2\left(x+y\right)\left(y+z\right)\left(z+x\right)}\)

Mặt khác:

\(\left(x+y\right)\left(y+z\right)\left(z+x\right)=\left(xy+yz+zx\right)\left(x+y+z\right)-xyz\)

\(=\left(x+y+z\right)\left(xy+yz+zx\right)-\sqrt[3]{xyz}.\sqrt[3]{xy.yz.zx}\)

\(\ge\left(x+y+z\right)\left(xy+yz+zx\right)-\dfrac{1}{3}.\left(x+y+z\right).\dfrac{1}{3}\left(xy+yz+zx\right)\)

\(=\dfrac{8}{9}\left(x+y+z\right)\left(zy+yz+zx\right)=\dfrac{8}{3}\left(xy+yz+zx\right)\)

\(\Rightarrow P\le\dfrac{xy+yz+zx}{2.\dfrac{8}{3}\left(xy+yz+zx\right)}=\dfrac{3}{16}\)

Dấu "=" xảy ra khi \(x=y=z=1\)

nhận ra là bài này sai đề :)))

26 tháng 1 2018

Bài 1

M=2x+y+z−15x+x+2y+z−15y+x+y+2z−15z

M=x+12−15x+y+12−15y+z+12−15z

M=x−3x+y−3y+z−3z

M=1−3x+1−3y+1−3z

M=3−(3x+3y+3z)

M=3−3(1x+1y+1z)

Áp dụng bất đẳng thức Cauchy - Schwarz dạng phân thức

⇒1x+1y+1z≥(1+1+1)2x+y+z=9x+y+z=34

⇒3(1x+1y+1z)≥94

⇒3−3(1x+1y+1z)≤34

⇔M≤34

Vậy M max=34

Dấu " = " xảy ra khi x=y=z=4

Bai nay tim GTLN moi dung nha

10 tháng 6 2023

Ta cần chứng minh: 

\(\dfrac{1}{a+b}\le\dfrac{1}{4}\left(\dfrac{1}{a}+\dfrac{1}{b}\right)\left(1\right)\left(a,b>0\right)\)

\(\Leftrightarrow\dfrac{4}{a+b}\le\dfrac{a+b}{ab}\\ \Leftrightarrow4ab\le\left(a+b\right)^2\\ \Leftrightarrow\left(a-b\right)^2\ge0\left(luôn.đúng\right)\)

\(DBXR\Leftrightarrow a=b\)

Do các phép biến đổi tương đương nên (1) luôn đúng

Áp dụng (1), ta có:

\(\dfrac{1}{2x+y+z}\le\dfrac{1}{4}\left(\dfrac{1}{x+y}+\dfrac{1}{x+z}\right)\le\dfrac{1}{4}\left[\dfrac{1}{4}\left(\dfrac{1}{x}+\dfrac{1}{y}\right)+\dfrac{1}{4}\left(\dfrac{1}{x}+\dfrac{1}{z}\right)\right]=\dfrac{1}{16}\left(\dfrac{2}{x}+\dfrac{1}{y}+\dfrac{1}{z}\right)\)

Chứng minh tương tự, ta được:

\(\dfrac{1}{x+2y+z}\le\dfrac{1}{16}\left(\dfrac{1}{x}+\dfrac{2}{y}+\dfrac{1}{z}\right)\)

\(\dfrac{1}{x+y+2z}\le\dfrac{1}{16}\left(\dfrac{1}{x}+\dfrac{1}{y}+\dfrac{2}{z}\right)\)

Cộng từng vế BĐT, ta được:

\(\dfrac{1}{2x+y+z}+\dfrac{1}{x+2y+z}+\dfrac{1}{x+y+2z}\le\dfrac{1}{16}.\left(\dfrac{4}{x}+\dfrac{4}{y}+\dfrac{4}{z}\right)=\dfrac{1}{4}.\left(\dfrac{1}{x}+\dfrac{1}{y}+\dfrac{1}{z}\right)=\dfrac{1}{4}.4=1\)Hay \(\dfrac{1}{2x+y+z}+\dfrac{1}{x+2y+z}+\dfrac{1}{x+y+2z}\le1\left(đpcm\right)\)

\(DBXR\Leftrightarrow x=y=z=\dfrac{3}{4}\)

10 tháng 6 2023

thank

13 tháng 8 2021

Bổ đề:\(\dfrac{1}{x}+\dfrac{1}{y}\ge\dfrac{4}{x+y}\Leftrightarrow\dfrac{1}{x+y}\le\dfrac{1}{4}\left(\dfrac{1}{x}+\dfrac{1}{y}\right)\)

Ta có:\(\dfrac{1}{2x+y+z}\le\dfrac{1}{4}\left(\dfrac{1}{x+y}+\dfrac{1}{x+z}\right)\le\dfrac{1}{4}.\dfrac{1}{4}\left(\dfrac{1}{x}+\dfrac{1}{y}+\dfrac{1}{x}+\dfrac{1}{z}\right)\)

Tương tự ta có:\(\dfrac{1}{2y+z+x}\le\dfrac{1}{4}.\dfrac{1}{4}\left(\dfrac{1}{y}+\dfrac{1}{z}+\dfrac{1}{y}+\dfrac{1}{x}\right)\)

                         \(\dfrac{1}{2z+x+y}\le\dfrac{1}{4}.\dfrac{1}{4}\left(\dfrac{1}{z}+\dfrac{1}{x}+\dfrac{1}{z}+\dfrac{1}{y}\right)\)

Cộng vế với vế ta có:

\(\dfrac{1}{2x+y+z}+\dfrac{1}{2y+z+x}+\dfrac{1}{2z+x+y}\le\dfrac{1}{16}\left[4\left(\dfrac{1}{x}+\dfrac{1}{y}+\dfrac{1}{z}\right)\right]=\dfrac{1}{16}.4.4=1\)

Dấu "=" xảy ra ⇔ \(x=y=z=\dfrac{3}{4}\)